ECON PLEASE HELP

Lakukan tugas rumah & ujian kamu dengan baik sekarang menggunakan Quizwiz!

Refer to Figure 6-22. The effective price sellers receive after the tax is imposed is a. $3.00. b. $2.00. c. $3.50. d. $5.00.

a. $3.00.

Refer to Figure 6-26. The effective price received by sellers after the tax is imposed is a. $8. b. $12. c. $14. d. $16.

a. $8.

Refer to Figure 5-3. Which demand curve is unit elastic? a. D b. None of the above. c. A d. B

b. None of the above.

In a market economy, who or what determines who produces each good and how much is produced? a. lotteries b. lawyers c. prices d. the government

c. prices

Refer to Figure 2-17. Point B represents an inefficient outcome for this economy. a. True b. False

false

Refer to Figure 4-18. Equilibrium price and quantity are, respectively, a. $15 and 200 units. b. $25 and 600 units. c. $35 and 200 units. d. $25 and 400 units.

$25 and 400 units.

In the economy of Talikastan in 2015, consumption was $1000, GDP was $1950, government purchases were $500, and investment was $700. What were Talikastan's net exports in 2015? a. $250 b. $2200 c. -$250 d. Net exports cannot be calculated from the information given.

-$250

Refer to Figure 2-5. The opportunity cost of obtaining 20 additional dryers by moving from point D to point A is a. None of the above; the economy cannot move from point D to point A. b. 0 washers. c. 40 washers. d. 20 washers.

0 washers.

Suppose that you have received $300 as a birthday gift. You can spend it today or you can put the money in a bank account for a year and earn 5 percent interest. The opportunity cost of spending the money today, in terms of what you could have after one year, is a. $0. b. $315. c. $305. d. $15.

315

The opportunity cost of moving from point U to point R is a. 80 bushels of apples. b. 60 bushels of apples. c. 160 sweaters. d. 80 sweaters.

60 bushels of apples

If macaroni and cheese is an inferior good, what would happen to the equilibrium price and quantity of macaroni and cheese if consumers' incomes rise? a. The equilibrium price would decrease, and the equilibrium quantity would increase. b. The equilibrium price would increase, and the equilibrium quantity would decrease. c. Both the equilibrium price and quantity would increase. d. Both the equilibrium price and quantity would decrease.

Both the equilibrium price and quantity would decrease.

Refer to Figure 5-2. As price falls from Pa to Pb, we could use the three demand curves to calculate three different values of the price elasticity of demand. Which of the three demand curves would produce the smallest elasticity? a. D1 b. All of the above are equally elastic. c. D3 d. D2

D3

Economists believe that production possibilities frontiers rarely have a bowed shape. a. True b. False

False

If a line passes through the points (20,5) and (10,10), then the slope of the line is -2. a. True b. False

False

Public service announcements, mandatory health warnings on cigarette packages, and the prohibition of cigarette advertising on television are all policies aimed at shifting the demand curve for cigarettes to the right. a. True b. False

False

The slope of a horizontal line is infinite, and the slope of a vertical line is zero. a. True b. False

False

Refer to Table 5-9. Along which of the supply curves does quantity supplied move proportionately more than the price? a. along supply curve B only b. None. Quantity supplied moves proportionately less than the price along all of the three supply curves. c. along supply curves B and C d. along all three supply curves

None. Quantity supplied moves proportionately less than the price along all of the three supply curves.

Refer to Figure 5-19. Which of the following statements is not correct? a. Supply curve C is unit elastic. b. Supply curve D is more elastic than supply curve C. c. Supply curve A is perfectly inelastic. d. Supply curve B is perfectly elastic.

Supply curve C is unit elastic.

What would happen to the equilibrium price and quantity of lattés if the cost to produce steamed milk, which is used to make lattés, increased, and scientists discovered that lattés cause heart attacks? a. Both the equilibrium price and quantity would increase. b. Both the equilibrium price and quantity would decrease. c. The equilibrium price would decrease, and the effect on equilibrium quantity would be ambiguous. d. The equilibrium quantity would decrease, and the effect on equilibrium price would be ambiguous.

The equilibrium quantity would decrease, and the effect on equilibrium price would be ambiguous.

For a particular good, a 3 percent increase in price causes a 10 percent decrease in quantity demanded. Which of the following statements is most likely applicable to this good? a. The relevant time horizon is short. b. The market for the good is broadly defined. c. The good is a necessity. d. There are many close substitutes for this good.

There are many close substitutes for this good.

A production point is said to be efficient if there is no way for the economy to produce more of one good without producing less of another. a. True b. False

True

All scientific models, including economic models, simplify reality in order to improve our understanding of it. a. True b. False

True

In the circular-flow diagram, households and firms are the decision makers. a. True b. False

True

Refer to Figure 2-14. Unemployment could cause this economy to produce at point B. a. True b. False

True

The concept of slope can be used to answer questions about how much one variable responds to changes in another variable. a. True b. False

True

The production possibilities frontier is a graph that shows the various combinations of outputs that the economy can possibly produce given the available factors of production and the available production technology. a. True b. False Hide Feedback

True

When quantity demanded exceeds quantity supplied at the current market price, the market has a shortage, and market price will likely rise in the future to eliminate the shortage. a. True b. False

True

A linear, upward-sloping supply curve has a. both a changing slope and a changing price elasticity of supply. b. both a constant slope and a constant price elasticity of supply. c. a constant slope and a changing price elasticity of supply. d. a changing slope and a constant price elasticity of supply.

a constant slope and a changing price elasticity of supply.

In the circular-flow diagram, households and firms are the decision makers. a. True b. False

a true

If the price elasticity of supply is 1.2, and a price increase led to a 5% increase in quantity supplied, then the price increase is about a. 4.2%. b. 6.2%. c. 0.24%. d. 6%. Hide Feedback

a. 4.2%.

Which of the following was not a reason OPEC failed to keep the price of oil high? a. The agreement OPEC members signed allowed each country to produce as much oil as each wanted. b. Over the long run, producers of oil outside of OPEC responded to higher prices by increasing oil exploration and by building new extraction capacity. c. Consumers replaced old inefficient cars with newer efficient ones. d. Consumers responded to higher prices with greater conservation.

a. The agreement OPEC members signed allowed each country to produce as much oil as each wanted.

Which of the following events would cause the price of oranges to fall? a. The price of land throughout Florida decreases, and Florida produces a significant proportion of the nation's oranges. b. All of the above are correct. c. There is a shortage of oranges. d. The FDA announces that bananas cause strokes, and oranges and bananas are substitutes. Hide Feedback

a. The price of land throughout Florida decreases, and Florida produces a significant proportion of the nation's oranges.

All scientific models, including economic models, simplify reality in order to improve our understanding of it. a. True b. False

a. True

The law of demand is true for most goods in the economy. a. True b. False

a. True

Net exports equal a. Y - (C + I + G). b. Y - (C - I - G). c. imports minus exports. d. exports plus imports.

a. Y - (C + I + G).

When a tax is imposed on the buyers of a good, the demand curve shifts a. downward by the amount of the tax. b. downward by less than the amount of the tax. c. upward by less than the amount of the tax. d. upward by the amount of the tax.

a. downward by the amount of the tax.

A demand curve displaying the relationship between the price of cars and the quantity demanded of cars should have a slope that is a. less than 0. b. between one and infinity. c. between zero and 1. d. undefined.

a. less than 0.

Refer to Figure 4-20. If the price is $10, then there would be a a. shortage of 600 units, and price would rise. b. surplus of 600 units, and price would rise. c. surplus of 400 units, and price would rise. d. shortage of 400 units, and price would rise. Hide Feedback

a. shortage of 600 units, and price would rise.

In the circular-flow diagram, a. the factors of production are labor, land, and capital. b. firms own the factors of production. c. the factors of production are also called "output." d. All of the above are correct.

a. the factors of production are labor, land, and capital.

In the circular-flow diagram, in the markets for a. the factors of production, households are sellers and firms are buyers. b. goods and services, households are sellers and firms are buyers. c. goods and services, households and firms are both sellers. d. the factors of production, households and firms are both buyers.

a. the factors of production, households are sellers and firms are buyers

In economics, the cost of something is a. what you give up to get it. b. often impossible to quantify, even in principle. c. always measured in units of time given up to get it. d. the dollar amount of obtaining it.

a. what you give up to get it.

Rent controls can cause a. ​all of these are possible results of rent controls. b. ​a decline in the quality of housing available for rent. c. ​the development of a black market to allocate apartments to renters. d. ​longer search times for renters attempting to locate an apartment.

a. ​all of these are possible results of rent controls.

If nominal GDP is $10 trillion and real GDP is $12 trillion, then the GDP deflator is a. 120, and this indicates that the price level has increased by 20 percent since the base year. b. 83.33, and this indicates that the price level has decreased by 16.67 percent since the base year. c. 83.33, and this indicates that the price level has increased by 83.33 percent since the base year. d. 120, and this indicates that the price level has increased by 120 percent since the base year.

b. 83.33, and this indicates that the price level has decreased by 16.67 percent since the base year.

If a binding price ceiling is imposed on the baby formula market, then a. the quantity of baby formula demanded will increase. b. All of the above are correct. c. the quantity of baby formula supplied will decrease. d. a shortage of baby formula will develop.

b. All of the above are correct.

What would happen to the equilibrium price and quantity of peanut butter if the price of peanuts went up, the price of jelly fell, fewer firms decided to produce peanut butter, and health officials announced that eating peanut butter was good for you? a. Price will fall, and the effect on quantity is ambiguous. b. Price will rise, and the effect on quantity is ambiguous. c. Quantity will rise, and the effect on price is ambiguous. d. Quantity will fall, and the effect on price is ambiguous.

b. Price will rise, and the effect on quantity is ambiguous.

Refer to Figure 2-4. This economy cannot produce at which point(s)? a. S b. Q, S c. Q d. Q, S, T

b. Q, S

The slope of a horizontal line is infinite, and the slope of a vertical line is zero. a. True b. False

b. false

Which of the following is likely to have the most price inelastic demand? a. diamond earrings b. gasoline in the short run c. strawberry-banana milk shakes d. box seats at a major league baseball game

b. gasoline in the short run

Congress intended that a. the entire FICA tax be paid by firms. b. half the FICA tax be paid by workers, and half be paid by firms. c. one-quarter of the FICA tax be paid by workers, and three-quarters be paid by firms. d. the entire FICA tax be paid by workers.

b. half the FICA tax be paid by workers, and half be paid by firms.

Refer to Figure 4-22. At a price of $20, there is a a. shortage of 4 units. b. surplus of 4 units. c. surplus of 8 units. d. shortage of 8 units.

b. surplus of 4 units.

Refer to Figure 2-8. Point K represents an outcome in which a. production is inefficient. b. the economy is using all of its resources to produce hammers. c. some of the economy's resources are unemployed. d. the economy is using all of its nails to produce hammers.

b. the economy is using all of its resources to produce hammers.

Years ago, thousands of country music fans risked their lives by rushing to buy tickets for a Willie Nelson concert at Carnegie Hall. This behavior indicates a. the ticket price was above the equilibrium price. b. the ticket price was below the equilibrium price. c. the ticket price was at the equilibrium price. d. nothing about the equilibrium price.

b. the ticket price was below the equilibrium price.

Refer to Figure 4-25. All else equal, an increase in the use of laptop computers for note-taking would cause a move from a. x to y. b. y to x. c. SA to SB. d. SB to SA.

b. y to x.

Refer to Figure 6-19. Suppose a tax of $2 per unit is imposed on this market. How much will buyers pay per unit after the tax is imposed? a. $7 b. between $5 and $7 c. between $3 and $5 d. $3

between $5 and $7

Suppose the demand for macaroni is inelastic, the supply of macaroni is elastic, the demand for cigarettes is inelastic, and the supply of cigarettes is elastic. If a tax were levied on the sellers of both of these commodities, we would expect that the burden of a. both taxes would fall more heavily on the buyers than on the sellers. b. the macaroni tax would fall more heavily on the buyers than on the sellers, and the burden of the cigarette tax would fall more heavily on the sellers than on the buyers. c. both taxes would fall more heavily on the sellers than on the buyers. d. the macaroni tax would fall more heavily on the sellers than on the buyers, and the burden of the cigarette tax would fall more heavily on the buyers than on the sellers.

both taxes would fall more heavily on the buyers than on the sellers.

Economists view normative statements as a. statements about the normal condition of the world. b. descriptive, making a claim about how the world is. c. prescriptive, making a claim about how the world ought to be. d. pessimistic, putting the worst possible interpretation on things.

c . prescriptive, making a claim about how the world ought to be.

Refer to Figure 2-5. The opportunity cost of obtaining 20 additional dryers by moving from point D to point A is a. 20 washers. b. 40 washers. c. 0 washers. d. None of the above; the economy cannot move from point D to point A.

c. 0 washers.

If a binding price floor is imposed on the video game market, then a. the quantity of video games supplied will increase. b. a surplus of video games will develop. c. All of the above are correct. d. the quantity of video games demanded will decrease.

c. All of the above are correct.

Suppose the equilibrium price of a tube of toothpaste is $2, and the government imposes a price floor of $3 per tube. As a result of the price floor, a. quantity supplied increases. b. quantity demanded decreases. c. All of the above are correct. d. there is a surplus.

c. All of the above are correct.

Refer to Figure 2-4. Efficient production is represented by which point(s)? a. R, T, U b. S, T c. R, U d. Q, R, U

c. R, U

Refer to Figure 2-14. Which combination of points show production possibilities only achievable with improvements in technology or increases in resources? a. Q, R, U, and V b. None of the above is correct. c. S and X d. T and W

c. S and X

Minimum-wage laws dictate a. the exact wage that firms must pay workers. b. a maximum wage that firms may pay workers. c. a minimum wage that firms may pay workers. d. both a minimum wage and a maximum wage that firms may pay workers.

c. a minimum wage that firms may pay workers.

Price ceilings and price floors that are binding a. are desirable because they make markets more efficient and more fair. b. can have the effect of restoring a market to equilibrium. c. cause surpluses and shortages to persist because price cannot adjust to the market equilibrium price. d. are imposed because they can make the poor in the economy better off without causing adverse effects.

c. cause surpluses and shortages to persist because price cannot adjust to the market equilibrium price.

Holding all other forces constant, if increasing the price of a good leads to a decrease in total revenue, then the demand for the good must be a. None of the above is correct because a price increase always leads to an decrease in total revenue. b. unit elastic. c. elastic. d. inelastic.

c. elastic.

When demand is perfectly inelastic, the price elasticity of demand a. approaches infinity, and the demand curve is horizontal. b. is zero, and the demand curve is horizontal. c. is zero, and the demand curve is vertical. d. approaches infinity, and the demand curve is vertical.

c. is zero, and the demand curve is vertical.

A perfectly inelastic demand implies that buyers a. respond substantially to an increase in price. b. decrease their purchases when the price rises. c. purchase the same amount as before when the price rises or falls. d. increase their purchases only slightly when the price falls.

c. purchase the same amount as before when the price rises or falls.

In the circular-flow diagram, in the markets for a. goods and services, households and firms are both sellers. b. goods and services, households are sellers and firms are buyers. c. the factors of production, households are sellers and firms are buyers. d. the factors of production, households and firms are both buyers. Hide Feedback

c. the factors of production, households are sellers and firms are buyers.

A U.S. firm produces nail guns in the first quarter of 2010 and adds them to its inventory. In the second quarter of 2010 the firm sells the nail guns to a U.S. construction company. In which quarter(s) does(do) these transactions raise investment? a. neither the first nor the second b. the first and the second c. the first but not the second d. the second but not the first

c. the first but not the second

A legal maximum on the price at which a good can be sold is called a price a. floor. b. subsidy. c. ceiling. d. support.

ceiling.

If a firm is a price taker, it operates in a a. monopolistically competitive market. b. oligopoly market. c. monopoly market. d. competitive market.

competitive market

Which of the following is the correct formula for the GDP deflator? a. [(nominal GDP - real GDP) /real GDP] x 100 b. [(GDP - GDP per person) / GDP per person] x 100 c. (real GDP/nominal GDP) x 100 d. (nominal GDP/real GDP) x 100

d. (nominal GDP/real GDP) x 100

When a tax is placed on the sellers of a product, the a. supply of the product decreases. b. size of the market decreases. c. effective price received by sellers decreases, and the price paid by buyers increases. d. All of the above are correct.

d. All of the above are correct.

If macaroni and cheese is an inferior good, what would happen to the equilibrium price and quantity of macaroni and cheese if consumers' incomes rise? a. The equilibrium price would decrease, and the equilibrium quantity would increase. b. The equilibrium price would increase, and the equilibrium quantity would decrease. c. Both the equilibrium price and quantity would increase. d. Both the equilibrium price and quantity would decrease.

d. Both the equilibrium price and quantity would decrease.

Refer to Figure 4-20. At a price of $20, which of the following statements is not correct? a. The market is in equilibrium. b. The quantity of the good that is bought and sold is 600 units. c. There is no pressure for price to change. d. Equilibrium price is equal to equilibrium quantity.

d. Equilibrium price is equal to equilibrium quantity.

Refer to Figure 4-26. Which of the following movements would illustrate the effect in the market for orange juice of an announcement by the American Dental Association that orange juice erodes tooth enamel? a. Point C to Point D b. Point A to Point B c. Point A to Point D d. Point C to Point B

d. Point C to Point B

Refer to Figure 2-14. Which combination of points show production possibilities only achievable with improvements in technology or increases in resources? a. None of the above is correct. b. T and W c. Q, R, U, and V d. S and X

d. S and X

Refer to Table 4-12. If both members and non-members are allowed to purchase tickets to this year's celebrity golf tournament and the country club sets the ticket price at $30, then there will be a. 600 tickets unsold. b. a shortage of 300 tickets. c. 600 tickets sold. d. a surplus of 300 tickets.

d. a surplus of 300 tickets.

Consider the market for portable air conditioners in equilibrium. When a heat wave strikes the equilibrium price a. increases, and the equilibrium quantity decreases. b. and quantity both decrease. c. decreases, and the equilibrium quantity increases. d. and quantity both increase.

d. and quantity both increase.

Which of the following is likely to have the most price inelastic demand? a. leather boots b. optional textbooks c. tablet computers d. lightbulbs

d. lightbulbs

The price elasticity of demand measures the a. responsiveness of quantity demanded to a change in income. b. size of the shortage created by the increase in demand. c. direction of the shift in the demand curve in response to a market event. d. magnitude of the response in quantity demanded to a change in price.

d. magnitude of the response in quantity demanded to a change in price.

Refer to Figure 6-13. If the government imposes a price ceiling of $6 on this market, then there will be a. a shortage of 20 units. b. a shortage of 10 units. c. a shortage of 5 units. d. no shortage.

d. no shortage.

Economists view normative statements as a. descriptive, making a claim about how the world is. b. statements about the normal condition of the world. c. pessimistic, putting the worst possible interpretation on things. d. prescriptive, making a claim about how the world ought to be.

d. prescriptive, making a claim about how the world ought to be.

A decrease in demand will cause a decrease in price, which will cause a decrease in supply. a. True b. False

false

A surplus is the same as an excess demand. a. True b. False

false

Baseballs and baseball bats are substitute goods. a. True b. False

false

Economists believe that production possibilities frontiers rarely have a bowed shape. a. True b. False

false

Public service announcements, mandatory health warnings on cigarette packages, and the prohibition of cigarette advertising on television are all policies aimed at shifting the demand curve for cigarettes to the right. a. True b. False

false

When quantity supplied exceeds quantity demanded at the current market price, the market has a surplus, and market price will likely rise in the future to eliminate the surplus. a. True b. False

false

Joe and Fred are economists. Joe thinks that the wealthiest 10% of the US population should be taxed a rate higher than the rest of society because they can better afford it. Fred thinks that everyone should be taxed at the same rate because that is the fairest scenario and the wealthy should not be penalized for their success. In this example, Joe and Fred a. disagree about the validity of a positive theory. b. None of the above is correct. c. have different normative views about tax policy. d. must both be incorrect because tax policy is never that simple. Hide Feedback

have different normative views about tax policy.

Exceptionally favorable growing conditions in the vineyards of Napa Valley would cause a(n) a. decrease in the supply of wine, increasing price. b. increase in the demand for wine, increasing price. c. decrease in the demand for wine, decreasing price. d. increase in the supply of wine, decreasing price.

increase in the supply of wine, decreasing price.

Refer to Figure 5-7. For prices below $5, demand is price a. elastic, and lowering price will increase total revenue. b. inelastic, and lowering price will increase total revenue. c. elastic, and raising price will increase total revenue. d. inelastic, and raising price will increase total revenue.

inelastic, and raising price will increase total revenue.

Another term for factors of production is a. inputs. b. services. c. goods. d. output.

inputs

Suppose roses are currently selling for $20 per dozen, but the equilibrium price of roses is $30 per dozen. We would expect a a. shortage to exist and the market price of roses to increase. b. surplus to exist and the market price of roses to decrease. c. shortage to exist and the market price of roses to decrease. d. surplus to exist and the market price of roses to increase.

shortage to exist and the market price of roses to increase.

Suppose roses are currently selling for $40 per dozen, but the equilibrium price of roses is $30 per dozen. We would expect a a. shortage to exist and the market price of roses to increase. b. surplus to exist and the market price of roses to increase. c. shortage to exist and the market price of roses to decrease. d. surplus to exist and the market price of roses to decrease.

surplus to exist and the market price of roses to decrease. Hide Feedback

Assume the market for tennis balls is perfectly competitive. When one tennis ball producer exits the market, a. the price of tennis balls decreases. b. there is no longer a market for tennis balls. c. the price of tennis balls increases. d. the price of tennis balls does not change.

the price of tennis balls does not change.

A production point is said to be efficient if there is no way for the economy to produce more of one good without producing less of another. a. True b. False

true

Refer to Figure 2-14. Unemployment could cause this economy to produce at point B. a. True b. False

true

The production possibilities frontier shows the opportunity cost of one good as measured in terms of the other good. a. True b. False

true


Set pelajaran terkait

Pharmacology Respiratory Quiz Questions

View Set

Chapter 3 (3.1-3.2) quiz History

View Set

Chapter 1: Making Personal Wellness Choices and Changing Wellness Behaviors

View Set